How to find the sum of another series












0












$begingroup$


How do I find the sum of a series



$$sum_{1 leq x < y < z}^infty frac{1}{3^x4^y5^z}$$










share|cite|improve this question









$endgroup$












  • $begingroup$
    Try something along these lines.
    $endgroup$
    – Clement C.
    Dec 24 '18 at 20:52






  • 1




    $begingroup$
    Who DELETED math.stackexchange.com/questions/3050493??
    $endgroup$
    – cgiovanardi
    Dec 25 '18 at 15:24
















0












$begingroup$


How do I find the sum of a series



$$sum_{1 leq x < y < z}^infty frac{1}{3^x4^y5^z}$$










share|cite|improve this question









$endgroup$












  • $begingroup$
    Try something along these lines.
    $endgroup$
    – Clement C.
    Dec 24 '18 at 20:52






  • 1




    $begingroup$
    Who DELETED math.stackexchange.com/questions/3050493??
    $endgroup$
    – cgiovanardi
    Dec 25 '18 at 15:24














0












0








0


3



$begingroup$


How do I find the sum of a series



$$sum_{1 leq x < y < z}^infty frac{1}{3^x4^y5^z}$$










share|cite|improve this question









$endgroup$




How do I find the sum of a series



$$sum_{1 leq x < y < z}^infty frac{1}{3^x4^y5^z}$$







sequences-and-series






share|cite|improve this question













share|cite|improve this question











share|cite|improve this question




share|cite|improve this question










asked Dec 24 '18 at 20:49









Some studentSome student

62




62












  • $begingroup$
    Try something along these lines.
    $endgroup$
    – Clement C.
    Dec 24 '18 at 20:52






  • 1




    $begingroup$
    Who DELETED math.stackexchange.com/questions/3050493??
    $endgroup$
    – cgiovanardi
    Dec 25 '18 at 15:24


















  • $begingroup$
    Try something along these lines.
    $endgroup$
    – Clement C.
    Dec 24 '18 at 20:52






  • 1




    $begingroup$
    Who DELETED math.stackexchange.com/questions/3050493??
    $endgroup$
    – cgiovanardi
    Dec 25 '18 at 15:24
















$begingroup$
Try something along these lines.
$endgroup$
– Clement C.
Dec 24 '18 at 20:52




$begingroup$
Try something along these lines.
$endgroup$
– Clement C.
Dec 24 '18 at 20:52




1




1




$begingroup$
Who DELETED math.stackexchange.com/questions/3050493??
$endgroup$
– cgiovanardi
Dec 25 '18 at 15:24




$begingroup$
Who DELETED math.stackexchange.com/questions/3050493??
$endgroup$
– cgiovanardi
Dec 25 '18 at 15:24










2 Answers
2






active

oldest

votes


















4












$begingroup$

Write $i=x,,j=y-x,,k=z-y$ so your sum is $sum_{ijk}frac{1}{60^i 20^j 5^k}$, all indices starting at $1$. But $sum_ifrac{1}{n^i}=frac{1}{n-1}$, so the result is $frac{1}{59times 19times 4}$.






share|cite|improve this answer









$endgroup$





















    0












    $begingroup$

    Direct calculation through expansion:
    $$begin{align}sum_{1 leq x < y < z}^infty frac{1}{3^x4^y5^z}=
    &color{red}{frac13left[frac1{4^2}left(frac1{5^3}+frac1{5^4}+cdotsright)+frac1{4^3}left(frac1{5^4}+frac1{5^5}+cdotsright)+cdotsright]}+\
    +&color{blue}{frac1{3^2}left[frac1{4^3}left(frac1{5^4}+frac1{5^5}+cdotsright)+frac1{4^4}left(frac1{5^5}+frac1{5^6}+cdotsright)+cdotsright]}+cdots=\
    =&color{red}{frac13left[frac1{4^2}cdot frac{1}{4cdot 5^2}+frac1{4^3}cdot frac1{4cdot 5^3}+cdotsright]}+\
    +&color{blue}{frac1{3^2}left[frac1{4^3}cdot frac1{4cdot 5^3}+frac1{4^4}cdot frac1{4cdot 5^4}+cdotsright]}+cdots=\
    =&color{red}{frac13cdot frac{1}{4^3cdot 5^2}cdot frac{4cdot 5}{19}}+\
    +&color{blue}{frac1{3^2}cdot frac{1}{4^4cdot 5^3}cdot frac{4cdot 5}{19}}+cdots=\
    =&frac{1}{3cdot 4^2cdot 5cdot 19}cdot frac{3cdot 4cdot 5}{59}=\
    =&frac1{4cdot 19cdot 59}.end{align}$$






    share|cite|improve this answer









    $endgroup$














      Your Answer








      StackExchange.ready(function() {
      var channelOptions = {
      tags: "".split(" "),
      id: "69"
      };
      initTagRenderer("".split(" "), "".split(" "), channelOptions);

      StackExchange.using("externalEditor", function() {
      // Have to fire editor after snippets, if snippets enabled
      if (StackExchange.settings.snippets.snippetsEnabled) {
      StackExchange.using("snippets", function() {
      createEditor();
      });
      }
      else {
      createEditor();
      }
      });

      function createEditor() {
      StackExchange.prepareEditor({
      heartbeatType: 'answer',
      autoActivateHeartbeat: false,
      convertImagesToLinks: true,
      noModals: true,
      showLowRepImageUploadWarning: true,
      reputationToPostImages: 10,
      bindNavPrevention: true,
      postfix: "",
      imageUploader: {
      brandingHtml: "Powered by u003ca class="icon-imgur-white" href="https://imgur.com/"u003eu003c/au003e",
      contentPolicyHtml: "User contributions licensed under u003ca href="https://creativecommons.org/licenses/by-sa/3.0/"u003ecc by-sa 3.0 with attribution requiredu003c/au003e u003ca href="https://stackoverflow.com/legal/content-policy"u003e(content policy)u003c/au003e",
      allowUrls: true
      },
      noCode: true, onDemand: true,
      discardSelector: ".discard-answer"
      ,immediatelyShowMarkdownHelp:true
      });


      }
      });














      draft saved

      draft discarded


















      StackExchange.ready(
      function () {
      StackExchange.openid.initPostLogin('.new-post-login', 'https%3a%2f%2fmath.stackexchange.com%2fquestions%2f3051623%2fhow-to-find-the-sum-of-another-series%23new-answer', 'question_page');
      }
      );

      Post as a guest















      Required, but never shown

























      2 Answers
      2






      active

      oldest

      votes








      2 Answers
      2






      active

      oldest

      votes









      active

      oldest

      votes






      active

      oldest

      votes









      4












      $begingroup$

      Write $i=x,,j=y-x,,k=z-y$ so your sum is $sum_{ijk}frac{1}{60^i 20^j 5^k}$, all indices starting at $1$. But $sum_ifrac{1}{n^i}=frac{1}{n-1}$, so the result is $frac{1}{59times 19times 4}$.






      share|cite|improve this answer









      $endgroup$


















        4












        $begingroup$

        Write $i=x,,j=y-x,,k=z-y$ so your sum is $sum_{ijk}frac{1}{60^i 20^j 5^k}$, all indices starting at $1$. But $sum_ifrac{1}{n^i}=frac{1}{n-1}$, so the result is $frac{1}{59times 19times 4}$.






        share|cite|improve this answer









        $endgroup$
















          4












          4








          4





          $begingroup$

          Write $i=x,,j=y-x,,k=z-y$ so your sum is $sum_{ijk}frac{1}{60^i 20^j 5^k}$, all indices starting at $1$. But $sum_ifrac{1}{n^i}=frac{1}{n-1}$, so the result is $frac{1}{59times 19times 4}$.






          share|cite|improve this answer









          $endgroup$



          Write $i=x,,j=y-x,,k=z-y$ so your sum is $sum_{ijk}frac{1}{60^i 20^j 5^k}$, all indices starting at $1$. But $sum_ifrac{1}{n^i}=frac{1}{n-1}$, so the result is $frac{1}{59times 19times 4}$.







          share|cite|improve this answer












          share|cite|improve this answer



          share|cite|improve this answer










          answered Dec 24 '18 at 20:54









          J.G.J.G.

          34.4k23252




          34.4k23252























              0












              $begingroup$

              Direct calculation through expansion:
              $$begin{align}sum_{1 leq x < y < z}^infty frac{1}{3^x4^y5^z}=
              &color{red}{frac13left[frac1{4^2}left(frac1{5^3}+frac1{5^4}+cdotsright)+frac1{4^3}left(frac1{5^4}+frac1{5^5}+cdotsright)+cdotsright]}+\
              +&color{blue}{frac1{3^2}left[frac1{4^3}left(frac1{5^4}+frac1{5^5}+cdotsright)+frac1{4^4}left(frac1{5^5}+frac1{5^6}+cdotsright)+cdotsright]}+cdots=\
              =&color{red}{frac13left[frac1{4^2}cdot frac{1}{4cdot 5^2}+frac1{4^3}cdot frac1{4cdot 5^3}+cdotsright]}+\
              +&color{blue}{frac1{3^2}left[frac1{4^3}cdot frac1{4cdot 5^3}+frac1{4^4}cdot frac1{4cdot 5^4}+cdotsright]}+cdots=\
              =&color{red}{frac13cdot frac{1}{4^3cdot 5^2}cdot frac{4cdot 5}{19}}+\
              +&color{blue}{frac1{3^2}cdot frac{1}{4^4cdot 5^3}cdot frac{4cdot 5}{19}}+cdots=\
              =&frac{1}{3cdot 4^2cdot 5cdot 19}cdot frac{3cdot 4cdot 5}{59}=\
              =&frac1{4cdot 19cdot 59}.end{align}$$






              share|cite|improve this answer









              $endgroup$


















                0












                $begingroup$

                Direct calculation through expansion:
                $$begin{align}sum_{1 leq x < y < z}^infty frac{1}{3^x4^y5^z}=
                &color{red}{frac13left[frac1{4^2}left(frac1{5^3}+frac1{5^4}+cdotsright)+frac1{4^3}left(frac1{5^4}+frac1{5^5}+cdotsright)+cdotsright]}+\
                +&color{blue}{frac1{3^2}left[frac1{4^3}left(frac1{5^4}+frac1{5^5}+cdotsright)+frac1{4^4}left(frac1{5^5}+frac1{5^6}+cdotsright)+cdotsright]}+cdots=\
                =&color{red}{frac13left[frac1{4^2}cdot frac{1}{4cdot 5^2}+frac1{4^3}cdot frac1{4cdot 5^3}+cdotsright]}+\
                +&color{blue}{frac1{3^2}left[frac1{4^3}cdot frac1{4cdot 5^3}+frac1{4^4}cdot frac1{4cdot 5^4}+cdotsright]}+cdots=\
                =&color{red}{frac13cdot frac{1}{4^3cdot 5^2}cdot frac{4cdot 5}{19}}+\
                +&color{blue}{frac1{3^2}cdot frac{1}{4^4cdot 5^3}cdot frac{4cdot 5}{19}}+cdots=\
                =&frac{1}{3cdot 4^2cdot 5cdot 19}cdot frac{3cdot 4cdot 5}{59}=\
                =&frac1{4cdot 19cdot 59}.end{align}$$






                share|cite|improve this answer









                $endgroup$
















                  0












                  0








                  0





                  $begingroup$

                  Direct calculation through expansion:
                  $$begin{align}sum_{1 leq x < y < z}^infty frac{1}{3^x4^y5^z}=
                  &color{red}{frac13left[frac1{4^2}left(frac1{5^3}+frac1{5^4}+cdotsright)+frac1{4^3}left(frac1{5^4}+frac1{5^5}+cdotsright)+cdotsright]}+\
                  +&color{blue}{frac1{3^2}left[frac1{4^3}left(frac1{5^4}+frac1{5^5}+cdotsright)+frac1{4^4}left(frac1{5^5}+frac1{5^6}+cdotsright)+cdotsright]}+cdots=\
                  =&color{red}{frac13left[frac1{4^2}cdot frac{1}{4cdot 5^2}+frac1{4^3}cdot frac1{4cdot 5^3}+cdotsright]}+\
                  +&color{blue}{frac1{3^2}left[frac1{4^3}cdot frac1{4cdot 5^3}+frac1{4^4}cdot frac1{4cdot 5^4}+cdotsright]}+cdots=\
                  =&color{red}{frac13cdot frac{1}{4^3cdot 5^2}cdot frac{4cdot 5}{19}}+\
                  +&color{blue}{frac1{3^2}cdot frac{1}{4^4cdot 5^3}cdot frac{4cdot 5}{19}}+cdots=\
                  =&frac{1}{3cdot 4^2cdot 5cdot 19}cdot frac{3cdot 4cdot 5}{59}=\
                  =&frac1{4cdot 19cdot 59}.end{align}$$






                  share|cite|improve this answer









                  $endgroup$



                  Direct calculation through expansion:
                  $$begin{align}sum_{1 leq x < y < z}^infty frac{1}{3^x4^y5^z}=
                  &color{red}{frac13left[frac1{4^2}left(frac1{5^3}+frac1{5^4}+cdotsright)+frac1{4^3}left(frac1{5^4}+frac1{5^5}+cdotsright)+cdotsright]}+\
                  +&color{blue}{frac1{3^2}left[frac1{4^3}left(frac1{5^4}+frac1{5^5}+cdotsright)+frac1{4^4}left(frac1{5^5}+frac1{5^6}+cdotsright)+cdotsright]}+cdots=\
                  =&color{red}{frac13left[frac1{4^2}cdot frac{1}{4cdot 5^2}+frac1{4^3}cdot frac1{4cdot 5^3}+cdotsright]}+\
                  +&color{blue}{frac1{3^2}left[frac1{4^3}cdot frac1{4cdot 5^3}+frac1{4^4}cdot frac1{4cdot 5^4}+cdotsright]}+cdots=\
                  =&color{red}{frac13cdot frac{1}{4^3cdot 5^2}cdot frac{4cdot 5}{19}}+\
                  +&color{blue}{frac1{3^2}cdot frac{1}{4^4cdot 5^3}cdot frac{4cdot 5}{19}}+cdots=\
                  =&frac{1}{3cdot 4^2cdot 5cdot 19}cdot frac{3cdot 4cdot 5}{59}=\
                  =&frac1{4cdot 19cdot 59}.end{align}$$







                  share|cite|improve this answer












                  share|cite|improve this answer



                  share|cite|improve this answer










                  answered Dec 25 '18 at 17:40









                  farruhotafarruhota

                  22.5k2942




                  22.5k2942






























                      draft saved

                      draft discarded




















































                      Thanks for contributing an answer to Mathematics Stack Exchange!


                      • Please be sure to answer the question. Provide details and share your research!

                      But avoid



                      • Asking for help, clarification, or responding to other answers.

                      • Making statements based on opinion; back them up with references or personal experience.


                      Use MathJax to format equations. MathJax reference.


                      To learn more, see our tips on writing great answers.




                      draft saved


                      draft discarded














                      StackExchange.ready(
                      function () {
                      StackExchange.openid.initPostLogin('.new-post-login', 'https%3a%2f%2fmath.stackexchange.com%2fquestions%2f3051623%2fhow-to-find-the-sum-of-another-series%23new-answer', 'question_page');
                      }
                      );

                      Post as a guest















                      Required, but never shown





















































                      Required, but never shown














                      Required, but never shown












                      Required, but never shown







                      Required, but never shown

































                      Required, but never shown














                      Required, but never shown












                      Required, but never shown







                      Required, but never shown







                      Popular posts from this blog

                      How to put 3 figures in Latex with 2 figures side by side and 1 below these side by side images but in...

                      In PowerPoint, is there a keyboard shortcut for bulleted / numbered list?

                      IC on Digikey is 5x more expensive than board containing same IC on Alibaba: How? [on hold]